Bạn chưa đăng nhập. Vui lòng đăng nhập để hỏi bài

Những câu hỏi liên quan
Bùi Hoàng Tuấn Kiệt
Xem chi tiết
Bùi Võ Đức Trọng
21 tháng 7 2021 lúc 14:05

đúng thì like giúp mik nha bạn. Thx bạnundefined

Chuyengia247
Xem chi tiết
Nguyễn Việt Lâm
20 tháng 1 2022 lúc 15:20

Do \(x^2+y^2=1\Rightarrow-1\le x;y\le1\Rightarrow\left\{{}\begin{matrix}y+1\ge0\\1-y\ge0\end{matrix}\right.\)

\(\Rightarrow\left\{{}\begin{matrix}y^2\left(y+1\right)\ge0\\y^2\left(1-y\right)\ge0\end{matrix}\right.\) \(\Rightarrow\left\{{}\begin{matrix}y^3\ge-y^2\\y^3\le y^2\end{matrix}\right.\)

Với mọi số thực x ta có:

\(\left\{{}\begin{matrix}\left(x+1\right)^2\ge0\\\left(x-1\right)^2\ge0\end{matrix}\right.\) \(\Rightarrow\left\{{}\begin{matrix}2x\ge-x^2-1\\2x\le x^2+1\end{matrix}\right.\)

Do đó: \(\left\{{}\begin{matrix}P=2x+y^3\ge-x^2-1-y^2=-2\\P=2x+y^3\le x^2+1+y^2=2\end{matrix}\right.\)

\(P_{min}=-2\) khi \(\left(x;y\right)=\left(-1;0\right)\)

\(P_{max}=2\) khi \(\left(x;y\right)=\left(1;0\right)\)

Bùi Hoàng Tuấn Kiệt
Xem chi tiết
Nguyễn Việt Lâm
21 tháng 7 2021 lúc 18:09

\(\dfrac{x^2+y^2}{2}\ge xy\Rightarrow-xy\ge-\dfrac{x^2+y^2}{2}\)

\(\Rightarrow4=x^2+y^2-xy\ge x^2+y^2-\dfrac{x^2+y^2}{2}=\dfrac{x^2+y^2}{2}\)

\(\Rightarrow x^2+y^2\le8\)

\(C_{max}=8\) khi \(x=y=\pm2\)

\(x^2+y^2\ge-2xy\Rightarrow-xy\le\dfrac{x^2+y^2}{2}\)

\(4=x^2+y^2-xy\le x^2+y^2+\dfrac{x^2+y^2}{2}=\dfrac{3}{2}\left(x^2+y^2\right)\)

\(\Rightarrow x^2+y^2\ge\dfrac{8}{3}\)

\(C_{min}=\dfrac{8}{3}\) khi \(\left(x;y\right)=\left(-\dfrac{2}{\sqrt{3}};\dfrac{2}{\sqrt{3}}\right);\left(\dfrac{2}{\sqrt{3}};-\dfrac{2}{\sqrt{3}}\right)\)

Bùi Võ Đức Trọng
21 tháng 7 2021 lúc 14:09

undefinedĐúng thì like giúp mik nha bạn. Thx bạn

Đỗ Gia Huy
Xem chi tiết
fan FA
13 tháng 8 2016 lúc 13:56

1. Đặt x = √2.cosα và y = √2.sinα (với α trên [0,3π/2]) 
Ta có: P = 4√2(sinα + cosα)(1 - sinαcosα) - 6sinαcosα 
Đặt t = sinα + cosα = √2.sin(α + π/4) có |t| ≤ √2, nên sinαcosα = (t^2 - 1)/2 
suy ra P = -2√2.t^3 - 3t^2 + 6√2.t + 3. 
Đến đây bạn áp dụng P' = 0 rồi xét các gtrị cực trị. 

2. Đặt x = cosα và y = sinα (với α trên [0,3π/2]) 
Biến đổi P = (6sin2α + cos2α + 1) / (3 + sin 2α - cos 2α) 
Mặt khác lại có (cos2α)^2 + (sin 2α)^2 = 1. 
Ta áp dụng P' = 0 tiếp.

I am➻Minh
Xem chi tiết
Phùng Minh Quân
30 tháng 7 2019 lúc 21:20

pt \(\Leftrightarrow\)\(\left(x+y\right)^2+7\left(x+y\right)+\frac{49}{4}=-y^2+\frac{49}{4}-10\)

\(\Leftrightarrow\)\(\left(x+y+\frac{7}{2}\right)^2=-y^2+\frac{9}{4}\le\frac{9}{4}\)

\(\Leftrightarrow\)\(\frac{-3}{2}\le x+y+\frac{7}{2}\le\frac{3}{2}\)

\(\Leftrightarrow\)\(-4\le x+y+1\le-1\)

Dấu "=" tự xét nhé 

Phạm Minh Quang
Xem chi tiết
Người Vô Danh
Xem chi tiết
Thảo Vũ
Xem chi tiết
Nguyễn Việt Lâm
23 tháng 12 2020 lúc 23:30

\(x^2+2xy+y^2+6\left(x+y\right)+8=-y^2\)

\(\Leftrightarrow\left(x+y\right)^2+6\left(x+y\right)+8\le0\)

\(\Leftrightarrow\left(x+y+2\right)\left(x+y+4\right)\le0\)

\(\Rightarrow-4\le x+y\le-2\)

\(\Rightarrow2016\le B\le2018\)

\(B_{min}=2016\) khi \(\left(x;y\right)=\left(-4;0\right)\)

\(B_{max}=2018\) khi \(\left(x;y\right)=\left(-2;0\right)\)

Anh VMin
Xem chi tiết
Đặng Tuấn Anh
13 tháng 4 2018 lúc 19:04

Ta có : \(2xy\le x^2+y^2=8\Rightarrow xy\le4\)

\(\Rightarrow x^2+y^2+2xy\le16\Leftrightarrow\left(x+y\right)^2\le4^2\Rightarrow-4\le x+y\le4\)

Vậy Max x+y là 4 khi x=y=2

       Min x+y là -4 khi x=y=-2